Rotation of Axes: Homework Statement & Equation Solution

  • Thread starter Thread starter Ledsnyder
  • Start date Start date
  • Tags Tags
    Axes Rotation
Click For Summary
The discussion focuses on transforming the equation y=10sech(x) after rotating the axes by 45 degrees. The transformation equations for individual points are provided, but the challenge lies in applying them to the entire equation. The participants suggest substituting the transformed coordinates into the original equation to derive the new equation in the rotated system. The final step involves eliminating X from the two transformed equations to find the new relationship in terms of Y. This process highlights the complexity of coordinate transformations in mathematical equations.
Ledsnyder
Messages
26
Reaction score
0

Homework Statement



THe x and y axes have been rotated about the origin through a 45 degree angle
to produce the X and Y axes.. Thus, a given point P (x,y) has coordinates in the first coordinate system and (X,Y) in the new coordinate system

The orignal equation is y=10sech(x).
What is the new equation in the new coordinate system?

Homework Equations





The Attempt at a Solution


x=  X cos(theta) - Y sin(theta)  y= X sin(theta)-  Y cos (theta)

These are for individual points but not for whole equations.
 
Physics news on Phys.org
Have you drawn the picture?
 
Ledsnyder said:

Homework Statement



THe x and y axes have been rotated about the origin through a 45 degree angle
to produce the X and Y axes.. Thus, a given point P (x,y) has coordinates in the first coordinate system and (X,Y) in the new coordinate system

The orignal equation is y=10sech(x).
What is the new equation in the new coordinate system?

Homework Equations


The Attempt at a Solution


x= X cos(theta) - Y sin(theta) y= X sin(theta)- Y cos (theta)

These are for individual points but not for whole equations.
And you are told that Y= sech(X) so those equations become
x= X cos(45)- sech(X)sin(45), y= X sin(45)- sech(X) cos (45).

Eliminate X from those two equations.
 
Question: A clock's minute hand has length 4 and its hour hand has length 3. What is the distance between the tips at the moment when it is increasing most rapidly?(Putnam Exam Question) Answer: Making assumption that both the hands moves at constant angular velocities, the answer is ## \sqrt{7} .## But don't you think this assumption is somewhat doubtful and wrong?

Similar threads

Replies
3
Views
2K
  • · Replies 1 ·
Replies
1
Views
2K
Replies
1
Views
2K
  • · Replies 3 ·
Replies
3
Views
2K
  • · Replies 4 ·
Replies
4
Views
2K
Replies
3
Views
2K
  • · Replies 5 ·
Replies
5
Views
953
  • · Replies 2 ·
Replies
2
Views
2K
  • · Replies 1 ·
Replies
1
Views
2K